When can a sum and integral be interchanged?












107












$begingroup$


Let's say I have $int_{0}^{infty}sum_{n = 0}^{infty} f_{n}(x), dx$ with $f_{n}(x)$ being continuous functions. When can interchange the integral and summation? Is $f_{n}(x) geq 0$ for all $x$ and for all $n$ sufficient? How about when $sum f_{n}(x)$ converges absolutely? If so why?










share|cite|improve this question









$endgroup$








  • 3




    $begingroup$
    I'm used to proving it capable with monotone convergence or the Lebesgue dominated convergence methods. But those are hardly sharp, I think. There are many versions of MC and LDC, so I don't know which you know.
    $endgroup$
    – davidlowryduda
    Nov 19 '11 at 19:21
















107












$begingroup$


Let's say I have $int_{0}^{infty}sum_{n = 0}^{infty} f_{n}(x), dx$ with $f_{n}(x)$ being continuous functions. When can interchange the integral and summation? Is $f_{n}(x) geq 0$ for all $x$ and for all $n$ sufficient? How about when $sum f_{n}(x)$ converges absolutely? If so why?










share|cite|improve this question









$endgroup$








  • 3




    $begingroup$
    I'm used to proving it capable with monotone convergence or the Lebesgue dominated convergence methods. But those are hardly sharp, I think. There are many versions of MC and LDC, so I don't know which you know.
    $endgroup$
    – davidlowryduda
    Nov 19 '11 at 19:21














107












107








107


87



$begingroup$


Let's say I have $int_{0}^{infty}sum_{n = 0}^{infty} f_{n}(x), dx$ with $f_{n}(x)$ being continuous functions. When can interchange the integral and summation? Is $f_{n}(x) geq 0$ for all $x$ and for all $n$ sufficient? How about when $sum f_{n}(x)$ converges absolutely? If so why?










share|cite|improve this question









$endgroup$




Let's say I have $int_{0}^{infty}sum_{n = 0}^{infty} f_{n}(x), dx$ with $f_{n}(x)$ being continuous functions. When can interchange the integral and summation? Is $f_{n}(x) geq 0$ for all $x$ and for all $n$ sufficient? How about when $sum f_{n}(x)$ converges absolutely? If so why?







real-analysis analysis






share|cite|improve this question













share|cite|improve this question











share|cite|improve this question




share|cite|improve this question










asked Nov 19 '11 at 19:08









user192837user192837

538254




538254








  • 3




    $begingroup$
    I'm used to proving it capable with monotone convergence or the Lebesgue dominated convergence methods. But those are hardly sharp, I think. There are many versions of MC and LDC, so I don't know which you know.
    $endgroup$
    – davidlowryduda
    Nov 19 '11 at 19:21














  • 3




    $begingroup$
    I'm used to proving it capable with monotone convergence or the Lebesgue dominated convergence methods. But those are hardly sharp, I think. There are many versions of MC and LDC, so I don't know which you know.
    $endgroup$
    – davidlowryduda
    Nov 19 '11 at 19:21








3




3




$begingroup$
I'm used to proving it capable with monotone convergence or the Lebesgue dominated convergence methods. But those are hardly sharp, I think. There are many versions of MC and LDC, so I don't know which you know.
$endgroup$
– davidlowryduda
Nov 19 '11 at 19:21




$begingroup$
I'm used to proving it capable with monotone convergence or the Lebesgue dominated convergence methods. But those are hardly sharp, I think. There are many versions of MC and LDC, so I don't know which you know.
$endgroup$
– davidlowryduda
Nov 19 '11 at 19:21










3 Answers
3






active

oldest

votes


















103












$begingroup$

I like to remember this as a special case of the Fubini/Tonelli theorems, where the measures are counting measure on $mathbb{N}$ and Lebesgue measure on $mathbb{R}$ (or $[0,infty)$ as you've written it here). In particular, Tonelli's theorem says if $f_n(x) ge 0$ for all $n,x$, then $$sum int f_n(x) dx = int sum f_n(x) dx$$ without any further conditions needed. (You can also prove this with the monotone convergence theorem.)



Then Fubini's theorem says that for general $f_n$, if $int sum |f_n| < infty$ or $sum int |f_n| < infty$ (by Tonelli the two conditions are equivalent), then $int sum f_n = sum int f_n$. (You can also prove this with the dominated convergence theorem.)



There may be weaker conditions that would also suffice, but these tend to work in 99% of cases.





Elaborating on request: the usual statement of Fubini's theorem goes something like this:




Let $(X,mathcal{F}, mu),(Y,mathcal{G}, nu)$ be $sigma$-finite measure spaces, and let $g : X times Y to mathbb{R}$ be measurable with respect to the product $sigma$-algebra $mathcal{F} otimes mathcal{G}$. Suppose that $int_X int_Y |g(x,y)| nu(dy) mu(dx)$ is finite. (Note: By Tonelli's theorem, this happens if and only if $int_Y int_X |g(x,y)|mu(dx)nu(dy)$ is finite, since both iterated integrals are equal.) Then $$int_X int_Y g(x,y) nu(dy)mu(dx) = int_Y int_X g(x,y) mu(dx) nu(dy).$$




Let $X = mathbb{R}$, $mathcal{F}$ the Borel $sigma$-algebra, and $mu$ Lebesgue measure. Let $Y = mathbb{N}$, $mathcal{G} = 2^{mathbb{N}}$ the discrete $sigma$-algebra, and $nu$ counting measure. Define $g(x,n) = f_n(x)$. Exercise: since each $f_n$ is measurable, verify that $g$ is measurable with respect to $mathcal{F} otimes mathcal{G}$. Exercise: verify that integration with respect to counting measure is the same as summation, where the integral exists and is finite iff the sum converges absolutely. (That is, given a sequence of real numbers $a_n$, define a function $b : mathbb{N} to mathbb{R}$ by $b(n) = a_n$. Then $int_{mathbb{N}} b,dnu = sum_{n=1}^infty a_n$.)



As such, the conclusion of Fubini's theorem reduces to the statement that was to be proved.






share|cite|improve this answer











$endgroup$













  • $begingroup$
    I am a bit confused because first you say that $sum int f_n(x) dx=int sum f_n(x) dx$ holds if $f_n(x) geq 0$which implies that even if the double integral is not finite the equality does hold and then you quote the Fubini theorem which says that the two integrals are equal if the double integral is finite. I would be grateful if you could explain this to me? Its probably as very stupid question but it makes me very uncomfortable
    $endgroup$
    – user3503589
    Nov 5 '14 at 9:24






  • 1




    $begingroup$
    Yes, that's what I said - I'm not sure what part is confusing you? Notice that I did not say "if and only if"! The two theorems give two different hypotheses, each of which leads to the same conclusion. (And note carefully the appearance of the absolute value bars in the hypothesis of Fubini's theorem.)
    $endgroup$
    – Nate Eldredge
    Nov 5 '14 at 14:48










  • $begingroup$
    Ah that was my confusion. I was thinking of the case when $f_n(x) geq 0$ and $int sum f_n(x) to infty$ and therefore Fubini theorem would be invalid(but luckily as you mentioned, its not"if and only if". Thank you for clearing it up Nate, though it was a stupid question.
    $endgroup$
    – user3503589
    Nov 6 '14 at 16:54










  • $begingroup$
    Nate: Can you please point out a standard reference for "Fubini's theorem says that for general $f_n$, if $int sum |f_n| < infty$ or $sum int |f_n| < infty$ (by Tonelli the two conditions are equivalent), then $int sum f_n = sum int f_n$"?
    $endgroup$
    – user389066
    Aug 15 '17 at 18:43












  • $begingroup$
    @User31443: Are you looking for a reference for the usual general statement of Fubini's theorem (Folland's Real Analysis, for instance) or for how the general statement implies this special case? It is literally just the general case applied with Lebesgue measure and counting measure.
    $endgroup$
    – Nate Eldredge
    Aug 15 '17 at 20:38



















25












$begingroup$

This is a theorem that will work:




Theorem. If ${f_n}_n$ is a positive sequence of integrable functions and $f = sum_n f_n$ then
$$int f = sum_n int f_n.$$




Proof. Consider first two functions, $f_1$ and $f_2$. We can now find sequences ${phi_j}_j$ and ${psi_j}_j$ of (nonnegative) simple functions by a basic theorem from measure theory that increase to $f_1$ and $f_2$ respectively. Obviously $phi_j + psi_j uparrow f_1 + f_2$. We can do the same for any finite sum.



Note that $int sum_1^N f_n = sum_1^N int f_n$ for any finite $N$. Now using the monotone convergence theorem we get



$$sum int f_n = int f.$$



Note 1: If you're talking about positive functions absolute convergence is the same as normal convergence as $|f_n| = f_n$.



Note 2: Continuous functions will be certainly integrable if they have compact support or tend to $0$ fast enough as $x to pm infty$.






share|cite|improve this answer











$endgroup$













  • $begingroup$
    Just a little observation: you don't have say that $phi_juparrow f_1$ and $psi_juparrow f_2$.
    $endgroup$
    – leo
    Nov 19 '11 at 19:44












  • $begingroup$
    @leo Thanks! I have added this.
    $endgroup$
    – Jonas Teuwen
    Nov 19 '11 at 19:46










  • $begingroup$
    No problem Jonas
    $endgroup$
    – leo
    Nov 19 '11 at 19:55






  • 1




    $begingroup$
    @JonasTeuwen What does $phi_j uparrow f_1$ mean? I am not familiar with this notation.
    $endgroup$
    – Noir
    Jun 17 '18 at 17:30










  • $begingroup$
    It means that ${phi_j}$ is a monotonically non-decreasing sequence of functions which converges to $f_1$ pointwise.
    $endgroup$
    – Sean Haight
    Oct 31 '18 at 15:40



















0












$begingroup$

While most of the time I would use the Fubini/Tonelli conditions, the dominated convergence theorem is actually strictly stronger in this mixed sum/integral case, because it can take into account the order structure of the integers. An example (that I first worked up back in 2009:



Consider the calculation
begin{align*}ln 2 &= int_0^1 frac1{1+x},dx = int_0^1sum_{n=0}^{infty} (-1)^n x^n,dx\
?&= sum_{n=0}^{infty}int_0^1(-1)^n x^n,dx = 1-frac12+frac13-frac14+cdotsend{align*}

Fubini's theorem isn't strong enough to justify the interchange. If we put absolute values on the terms, it blows up to $int_0^1 frac1{1-x},dx = 1+frac12+frac13+frac14+cdots=infty$.



On the other hand, the dominated convergence theorem cares about the partial sums $sum_{n=0}^{N}(-1)^n x^n$. By the alternating series estimate,
$$0le sum_{n=0}^{N}(-1)^n x^nle 1$$
for all $xin [0,1]$. $1$ is integrable on this interval, and the interchange
$$int_0^1left(lim_{Ntoinfty}sum_{n=0}^{N}(-1)^n x^nright),dx = lim_{Ntoinfty}int_0^1 sum_{n=0}^{N}(-1)^n x^n,dx$$
is justified, proving the result $1-frac12+frac13-frac14+cdots=ln 2$.



This situation with the dominated convergence theorem being stronger than Fubini's theorem can come up when we've got a reasonable bound on partial sums but not absolute convergence as a whole.

The monotone convergence theorem, on the other hand, is exactly the same as Tonelli's theorem - when everything's positive, either both sides are the same and finite or both sides are infinite.






share|cite|improve this answer









$endgroup$













    Your Answer





    StackExchange.ifUsing("editor", function () {
    return StackExchange.using("mathjaxEditing", function () {
    StackExchange.MarkdownEditor.creationCallbacks.add(function (editor, postfix) {
    StackExchange.mathjaxEditing.prepareWmdForMathJax(editor, postfix, [["$", "$"], ["\\(","\\)"]]);
    });
    });
    }, "mathjax-editing");

    StackExchange.ready(function() {
    var channelOptions = {
    tags: "".split(" "),
    id: "69"
    };
    initTagRenderer("".split(" "), "".split(" "), channelOptions);

    StackExchange.using("externalEditor", function() {
    // Have to fire editor after snippets, if snippets enabled
    if (StackExchange.settings.snippets.snippetsEnabled) {
    StackExchange.using("snippets", function() {
    createEditor();
    });
    }
    else {
    createEditor();
    }
    });

    function createEditor() {
    StackExchange.prepareEditor({
    heartbeatType: 'answer',
    autoActivateHeartbeat: false,
    convertImagesToLinks: true,
    noModals: true,
    showLowRepImageUploadWarning: true,
    reputationToPostImages: 10,
    bindNavPrevention: true,
    postfix: "",
    imageUploader: {
    brandingHtml: "Powered by u003ca class="icon-imgur-white" href="https://imgur.com/"u003eu003c/au003e",
    contentPolicyHtml: "User contributions licensed under u003ca href="https://creativecommons.org/licenses/by-sa/3.0/"u003ecc by-sa 3.0 with attribution requiredu003c/au003e u003ca href="https://stackoverflow.com/legal/content-policy"u003e(content policy)u003c/au003e",
    allowUrls: true
    },
    noCode: true, onDemand: true,
    discardSelector: ".discard-answer"
    ,immediatelyShowMarkdownHelp:true
    });


    }
    });














    draft saved

    draft discarded


















    StackExchange.ready(
    function () {
    StackExchange.openid.initPostLogin('.new-post-login', 'https%3a%2f%2fmath.stackexchange.com%2fquestions%2f83721%2fwhen-can-a-sum-and-integral-be-interchanged%23new-answer', 'question_page');
    }
    );

    Post as a guest















    Required, but never shown

























    3 Answers
    3






    active

    oldest

    votes








    3 Answers
    3






    active

    oldest

    votes









    active

    oldest

    votes






    active

    oldest

    votes









    103












    $begingroup$

    I like to remember this as a special case of the Fubini/Tonelli theorems, where the measures are counting measure on $mathbb{N}$ and Lebesgue measure on $mathbb{R}$ (or $[0,infty)$ as you've written it here). In particular, Tonelli's theorem says if $f_n(x) ge 0$ for all $n,x$, then $$sum int f_n(x) dx = int sum f_n(x) dx$$ without any further conditions needed. (You can also prove this with the monotone convergence theorem.)



    Then Fubini's theorem says that for general $f_n$, if $int sum |f_n| < infty$ or $sum int |f_n| < infty$ (by Tonelli the two conditions are equivalent), then $int sum f_n = sum int f_n$. (You can also prove this with the dominated convergence theorem.)



    There may be weaker conditions that would also suffice, but these tend to work in 99% of cases.





    Elaborating on request: the usual statement of Fubini's theorem goes something like this:




    Let $(X,mathcal{F}, mu),(Y,mathcal{G}, nu)$ be $sigma$-finite measure spaces, and let $g : X times Y to mathbb{R}$ be measurable with respect to the product $sigma$-algebra $mathcal{F} otimes mathcal{G}$. Suppose that $int_X int_Y |g(x,y)| nu(dy) mu(dx)$ is finite. (Note: By Tonelli's theorem, this happens if and only if $int_Y int_X |g(x,y)|mu(dx)nu(dy)$ is finite, since both iterated integrals are equal.) Then $$int_X int_Y g(x,y) nu(dy)mu(dx) = int_Y int_X g(x,y) mu(dx) nu(dy).$$




    Let $X = mathbb{R}$, $mathcal{F}$ the Borel $sigma$-algebra, and $mu$ Lebesgue measure. Let $Y = mathbb{N}$, $mathcal{G} = 2^{mathbb{N}}$ the discrete $sigma$-algebra, and $nu$ counting measure. Define $g(x,n) = f_n(x)$. Exercise: since each $f_n$ is measurable, verify that $g$ is measurable with respect to $mathcal{F} otimes mathcal{G}$. Exercise: verify that integration with respect to counting measure is the same as summation, where the integral exists and is finite iff the sum converges absolutely. (That is, given a sequence of real numbers $a_n$, define a function $b : mathbb{N} to mathbb{R}$ by $b(n) = a_n$. Then $int_{mathbb{N}} b,dnu = sum_{n=1}^infty a_n$.)



    As such, the conclusion of Fubini's theorem reduces to the statement that was to be proved.






    share|cite|improve this answer











    $endgroup$













    • $begingroup$
      I am a bit confused because first you say that $sum int f_n(x) dx=int sum f_n(x) dx$ holds if $f_n(x) geq 0$which implies that even if the double integral is not finite the equality does hold and then you quote the Fubini theorem which says that the two integrals are equal if the double integral is finite. I would be grateful if you could explain this to me? Its probably as very stupid question but it makes me very uncomfortable
      $endgroup$
      – user3503589
      Nov 5 '14 at 9:24






    • 1




      $begingroup$
      Yes, that's what I said - I'm not sure what part is confusing you? Notice that I did not say "if and only if"! The two theorems give two different hypotheses, each of which leads to the same conclusion. (And note carefully the appearance of the absolute value bars in the hypothesis of Fubini's theorem.)
      $endgroup$
      – Nate Eldredge
      Nov 5 '14 at 14:48










    • $begingroup$
      Ah that was my confusion. I was thinking of the case when $f_n(x) geq 0$ and $int sum f_n(x) to infty$ and therefore Fubini theorem would be invalid(but luckily as you mentioned, its not"if and only if". Thank you for clearing it up Nate, though it was a stupid question.
      $endgroup$
      – user3503589
      Nov 6 '14 at 16:54










    • $begingroup$
      Nate: Can you please point out a standard reference for "Fubini's theorem says that for general $f_n$, if $int sum |f_n| < infty$ or $sum int |f_n| < infty$ (by Tonelli the two conditions are equivalent), then $int sum f_n = sum int f_n$"?
      $endgroup$
      – user389066
      Aug 15 '17 at 18:43












    • $begingroup$
      @User31443: Are you looking for a reference for the usual general statement of Fubini's theorem (Folland's Real Analysis, for instance) or for how the general statement implies this special case? It is literally just the general case applied with Lebesgue measure and counting measure.
      $endgroup$
      – Nate Eldredge
      Aug 15 '17 at 20:38
















    103












    $begingroup$

    I like to remember this as a special case of the Fubini/Tonelli theorems, where the measures are counting measure on $mathbb{N}$ and Lebesgue measure on $mathbb{R}$ (or $[0,infty)$ as you've written it here). In particular, Tonelli's theorem says if $f_n(x) ge 0$ for all $n,x$, then $$sum int f_n(x) dx = int sum f_n(x) dx$$ without any further conditions needed. (You can also prove this with the monotone convergence theorem.)



    Then Fubini's theorem says that for general $f_n$, if $int sum |f_n| < infty$ or $sum int |f_n| < infty$ (by Tonelli the two conditions are equivalent), then $int sum f_n = sum int f_n$. (You can also prove this with the dominated convergence theorem.)



    There may be weaker conditions that would also suffice, but these tend to work in 99% of cases.





    Elaborating on request: the usual statement of Fubini's theorem goes something like this:




    Let $(X,mathcal{F}, mu),(Y,mathcal{G}, nu)$ be $sigma$-finite measure spaces, and let $g : X times Y to mathbb{R}$ be measurable with respect to the product $sigma$-algebra $mathcal{F} otimes mathcal{G}$. Suppose that $int_X int_Y |g(x,y)| nu(dy) mu(dx)$ is finite. (Note: By Tonelli's theorem, this happens if and only if $int_Y int_X |g(x,y)|mu(dx)nu(dy)$ is finite, since both iterated integrals are equal.) Then $$int_X int_Y g(x,y) nu(dy)mu(dx) = int_Y int_X g(x,y) mu(dx) nu(dy).$$




    Let $X = mathbb{R}$, $mathcal{F}$ the Borel $sigma$-algebra, and $mu$ Lebesgue measure. Let $Y = mathbb{N}$, $mathcal{G} = 2^{mathbb{N}}$ the discrete $sigma$-algebra, and $nu$ counting measure. Define $g(x,n) = f_n(x)$. Exercise: since each $f_n$ is measurable, verify that $g$ is measurable with respect to $mathcal{F} otimes mathcal{G}$. Exercise: verify that integration with respect to counting measure is the same as summation, where the integral exists and is finite iff the sum converges absolutely. (That is, given a sequence of real numbers $a_n$, define a function $b : mathbb{N} to mathbb{R}$ by $b(n) = a_n$. Then $int_{mathbb{N}} b,dnu = sum_{n=1}^infty a_n$.)



    As such, the conclusion of Fubini's theorem reduces to the statement that was to be proved.






    share|cite|improve this answer











    $endgroup$













    • $begingroup$
      I am a bit confused because first you say that $sum int f_n(x) dx=int sum f_n(x) dx$ holds if $f_n(x) geq 0$which implies that even if the double integral is not finite the equality does hold and then you quote the Fubini theorem which says that the two integrals are equal if the double integral is finite. I would be grateful if you could explain this to me? Its probably as very stupid question but it makes me very uncomfortable
      $endgroup$
      – user3503589
      Nov 5 '14 at 9:24






    • 1




      $begingroup$
      Yes, that's what I said - I'm not sure what part is confusing you? Notice that I did not say "if and only if"! The two theorems give two different hypotheses, each of which leads to the same conclusion. (And note carefully the appearance of the absolute value bars in the hypothesis of Fubini's theorem.)
      $endgroup$
      – Nate Eldredge
      Nov 5 '14 at 14:48










    • $begingroup$
      Ah that was my confusion. I was thinking of the case when $f_n(x) geq 0$ and $int sum f_n(x) to infty$ and therefore Fubini theorem would be invalid(but luckily as you mentioned, its not"if and only if". Thank you for clearing it up Nate, though it was a stupid question.
      $endgroup$
      – user3503589
      Nov 6 '14 at 16:54










    • $begingroup$
      Nate: Can you please point out a standard reference for "Fubini's theorem says that for general $f_n$, if $int sum |f_n| < infty$ or $sum int |f_n| < infty$ (by Tonelli the two conditions are equivalent), then $int sum f_n = sum int f_n$"?
      $endgroup$
      – user389066
      Aug 15 '17 at 18:43












    • $begingroup$
      @User31443: Are you looking for a reference for the usual general statement of Fubini's theorem (Folland's Real Analysis, for instance) or for how the general statement implies this special case? It is literally just the general case applied with Lebesgue measure and counting measure.
      $endgroup$
      – Nate Eldredge
      Aug 15 '17 at 20:38














    103












    103








    103





    $begingroup$

    I like to remember this as a special case of the Fubini/Tonelli theorems, where the measures are counting measure on $mathbb{N}$ and Lebesgue measure on $mathbb{R}$ (or $[0,infty)$ as you've written it here). In particular, Tonelli's theorem says if $f_n(x) ge 0$ for all $n,x$, then $$sum int f_n(x) dx = int sum f_n(x) dx$$ without any further conditions needed. (You can also prove this with the monotone convergence theorem.)



    Then Fubini's theorem says that for general $f_n$, if $int sum |f_n| < infty$ or $sum int |f_n| < infty$ (by Tonelli the two conditions are equivalent), then $int sum f_n = sum int f_n$. (You can also prove this with the dominated convergence theorem.)



    There may be weaker conditions that would also suffice, but these tend to work in 99% of cases.





    Elaborating on request: the usual statement of Fubini's theorem goes something like this:




    Let $(X,mathcal{F}, mu),(Y,mathcal{G}, nu)$ be $sigma$-finite measure spaces, and let $g : X times Y to mathbb{R}$ be measurable with respect to the product $sigma$-algebra $mathcal{F} otimes mathcal{G}$. Suppose that $int_X int_Y |g(x,y)| nu(dy) mu(dx)$ is finite. (Note: By Tonelli's theorem, this happens if and only if $int_Y int_X |g(x,y)|mu(dx)nu(dy)$ is finite, since both iterated integrals are equal.) Then $$int_X int_Y g(x,y) nu(dy)mu(dx) = int_Y int_X g(x,y) mu(dx) nu(dy).$$




    Let $X = mathbb{R}$, $mathcal{F}$ the Borel $sigma$-algebra, and $mu$ Lebesgue measure. Let $Y = mathbb{N}$, $mathcal{G} = 2^{mathbb{N}}$ the discrete $sigma$-algebra, and $nu$ counting measure. Define $g(x,n) = f_n(x)$. Exercise: since each $f_n$ is measurable, verify that $g$ is measurable with respect to $mathcal{F} otimes mathcal{G}$. Exercise: verify that integration with respect to counting measure is the same as summation, where the integral exists and is finite iff the sum converges absolutely. (That is, given a sequence of real numbers $a_n$, define a function $b : mathbb{N} to mathbb{R}$ by $b(n) = a_n$. Then $int_{mathbb{N}} b,dnu = sum_{n=1}^infty a_n$.)



    As such, the conclusion of Fubini's theorem reduces to the statement that was to be proved.






    share|cite|improve this answer











    $endgroup$



    I like to remember this as a special case of the Fubini/Tonelli theorems, where the measures are counting measure on $mathbb{N}$ and Lebesgue measure on $mathbb{R}$ (or $[0,infty)$ as you've written it here). In particular, Tonelli's theorem says if $f_n(x) ge 0$ for all $n,x$, then $$sum int f_n(x) dx = int sum f_n(x) dx$$ without any further conditions needed. (You can also prove this with the monotone convergence theorem.)



    Then Fubini's theorem says that for general $f_n$, if $int sum |f_n| < infty$ or $sum int |f_n| < infty$ (by Tonelli the two conditions are equivalent), then $int sum f_n = sum int f_n$. (You can also prove this with the dominated convergence theorem.)



    There may be weaker conditions that would also suffice, but these tend to work in 99% of cases.





    Elaborating on request: the usual statement of Fubini's theorem goes something like this:




    Let $(X,mathcal{F}, mu),(Y,mathcal{G}, nu)$ be $sigma$-finite measure spaces, and let $g : X times Y to mathbb{R}$ be measurable with respect to the product $sigma$-algebra $mathcal{F} otimes mathcal{G}$. Suppose that $int_X int_Y |g(x,y)| nu(dy) mu(dx)$ is finite. (Note: By Tonelli's theorem, this happens if and only if $int_Y int_X |g(x,y)|mu(dx)nu(dy)$ is finite, since both iterated integrals are equal.) Then $$int_X int_Y g(x,y) nu(dy)mu(dx) = int_Y int_X g(x,y) mu(dx) nu(dy).$$




    Let $X = mathbb{R}$, $mathcal{F}$ the Borel $sigma$-algebra, and $mu$ Lebesgue measure. Let $Y = mathbb{N}$, $mathcal{G} = 2^{mathbb{N}}$ the discrete $sigma$-algebra, and $nu$ counting measure. Define $g(x,n) = f_n(x)$. Exercise: since each $f_n$ is measurable, verify that $g$ is measurable with respect to $mathcal{F} otimes mathcal{G}$. Exercise: verify that integration with respect to counting measure is the same as summation, where the integral exists and is finite iff the sum converges absolutely. (That is, given a sequence of real numbers $a_n$, define a function $b : mathbb{N} to mathbb{R}$ by $b(n) = a_n$. Then $int_{mathbb{N}} b,dnu = sum_{n=1}^infty a_n$.)



    As such, the conclusion of Fubini's theorem reduces to the statement that was to be proved.







    share|cite|improve this answer














    share|cite|improve this answer



    share|cite|improve this answer








    edited Aug 15 '17 at 20:58

























    answered Nov 19 '11 at 21:32









    Nate EldredgeNate Eldredge

    62.6k682170




    62.6k682170












    • $begingroup$
      I am a bit confused because first you say that $sum int f_n(x) dx=int sum f_n(x) dx$ holds if $f_n(x) geq 0$which implies that even if the double integral is not finite the equality does hold and then you quote the Fubini theorem which says that the two integrals are equal if the double integral is finite. I would be grateful if you could explain this to me? Its probably as very stupid question but it makes me very uncomfortable
      $endgroup$
      – user3503589
      Nov 5 '14 at 9:24






    • 1




      $begingroup$
      Yes, that's what I said - I'm not sure what part is confusing you? Notice that I did not say "if and only if"! The two theorems give two different hypotheses, each of which leads to the same conclusion. (And note carefully the appearance of the absolute value bars in the hypothesis of Fubini's theorem.)
      $endgroup$
      – Nate Eldredge
      Nov 5 '14 at 14:48










    • $begingroup$
      Ah that was my confusion. I was thinking of the case when $f_n(x) geq 0$ and $int sum f_n(x) to infty$ and therefore Fubini theorem would be invalid(but luckily as you mentioned, its not"if and only if". Thank you for clearing it up Nate, though it was a stupid question.
      $endgroup$
      – user3503589
      Nov 6 '14 at 16:54










    • $begingroup$
      Nate: Can you please point out a standard reference for "Fubini's theorem says that for general $f_n$, if $int sum |f_n| < infty$ or $sum int |f_n| < infty$ (by Tonelli the two conditions are equivalent), then $int sum f_n = sum int f_n$"?
      $endgroup$
      – user389066
      Aug 15 '17 at 18:43












    • $begingroup$
      @User31443: Are you looking for a reference for the usual general statement of Fubini's theorem (Folland's Real Analysis, for instance) or for how the general statement implies this special case? It is literally just the general case applied with Lebesgue measure and counting measure.
      $endgroup$
      – Nate Eldredge
      Aug 15 '17 at 20:38


















    • $begingroup$
      I am a bit confused because first you say that $sum int f_n(x) dx=int sum f_n(x) dx$ holds if $f_n(x) geq 0$which implies that even if the double integral is not finite the equality does hold and then you quote the Fubini theorem which says that the two integrals are equal if the double integral is finite. I would be grateful if you could explain this to me? Its probably as very stupid question but it makes me very uncomfortable
      $endgroup$
      – user3503589
      Nov 5 '14 at 9:24






    • 1




      $begingroup$
      Yes, that's what I said - I'm not sure what part is confusing you? Notice that I did not say "if and only if"! The two theorems give two different hypotheses, each of which leads to the same conclusion. (And note carefully the appearance of the absolute value bars in the hypothesis of Fubini's theorem.)
      $endgroup$
      – Nate Eldredge
      Nov 5 '14 at 14:48










    • $begingroup$
      Ah that was my confusion. I was thinking of the case when $f_n(x) geq 0$ and $int sum f_n(x) to infty$ and therefore Fubini theorem would be invalid(but luckily as you mentioned, its not"if and only if". Thank you for clearing it up Nate, though it was a stupid question.
      $endgroup$
      – user3503589
      Nov 6 '14 at 16:54










    • $begingroup$
      Nate: Can you please point out a standard reference for "Fubini's theorem says that for general $f_n$, if $int sum |f_n| < infty$ or $sum int |f_n| < infty$ (by Tonelli the two conditions are equivalent), then $int sum f_n = sum int f_n$"?
      $endgroup$
      – user389066
      Aug 15 '17 at 18:43












    • $begingroup$
      @User31443: Are you looking for a reference for the usual general statement of Fubini's theorem (Folland's Real Analysis, for instance) or for how the general statement implies this special case? It is literally just the general case applied with Lebesgue measure and counting measure.
      $endgroup$
      – Nate Eldredge
      Aug 15 '17 at 20:38
















    $begingroup$
    I am a bit confused because first you say that $sum int f_n(x) dx=int sum f_n(x) dx$ holds if $f_n(x) geq 0$which implies that even if the double integral is not finite the equality does hold and then you quote the Fubini theorem which says that the two integrals are equal if the double integral is finite. I would be grateful if you could explain this to me? Its probably as very stupid question but it makes me very uncomfortable
    $endgroup$
    – user3503589
    Nov 5 '14 at 9:24




    $begingroup$
    I am a bit confused because first you say that $sum int f_n(x) dx=int sum f_n(x) dx$ holds if $f_n(x) geq 0$which implies that even if the double integral is not finite the equality does hold and then you quote the Fubini theorem which says that the two integrals are equal if the double integral is finite. I would be grateful if you could explain this to me? Its probably as very stupid question but it makes me very uncomfortable
    $endgroup$
    – user3503589
    Nov 5 '14 at 9:24




    1




    1




    $begingroup$
    Yes, that's what I said - I'm not sure what part is confusing you? Notice that I did not say "if and only if"! The two theorems give two different hypotheses, each of which leads to the same conclusion. (And note carefully the appearance of the absolute value bars in the hypothesis of Fubini's theorem.)
    $endgroup$
    – Nate Eldredge
    Nov 5 '14 at 14:48




    $begingroup$
    Yes, that's what I said - I'm not sure what part is confusing you? Notice that I did not say "if and only if"! The two theorems give two different hypotheses, each of which leads to the same conclusion. (And note carefully the appearance of the absolute value bars in the hypothesis of Fubini's theorem.)
    $endgroup$
    – Nate Eldredge
    Nov 5 '14 at 14:48












    $begingroup$
    Ah that was my confusion. I was thinking of the case when $f_n(x) geq 0$ and $int sum f_n(x) to infty$ and therefore Fubini theorem would be invalid(but luckily as you mentioned, its not"if and only if". Thank you for clearing it up Nate, though it was a stupid question.
    $endgroup$
    – user3503589
    Nov 6 '14 at 16:54




    $begingroup$
    Ah that was my confusion. I was thinking of the case when $f_n(x) geq 0$ and $int sum f_n(x) to infty$ and therefore Fubini theorem would be invalid(but luckily as you mentioned, its not"if and only if". Thank you for clearing it up Nate, though it was a stupid question.
    $endgroup$
    – user3503589
    Nov 6 '14 at 16:54












    $begingroup$
    Nate: Can you please point out a standard reference for "Fubini's theorem says that for general $f_n$, if $int sum |f_n| < infty$ or $sum int |f_n| < infty$ (by Tonelli the two conditions are equivalent), then $int sum f_n = sum int f_n$"?
    $endgroup$
    – user389066
    Aug 15 '17 at 18:43






    $begingroup$
    Nate: Can you please point out a standard reference for "Fubini's theorem says that for general $f_n$, if $int sum |f_n| < infty$ or $sum int |f_n| < infty$ (by Tonelli the two conditions are equivalent), then $int sum f_n = sum int f_n$"?
    $endgroup$
    – user389066
    Aug 15 '17 at 18:43














    $begingroup$
    @User31443: Are you looking for a reference for the usual general statement of Fubini's theorem (Folland's Real Analysis, for instance) or for how the general statement implies this special case? It is literally just the general case applied with Lebesgue measure and counting measure.
    $endgroup$
    – Nate Eldredge
    Aug 15 '17 at 20:38




    $begingroup$
    @User31443: Are you looking for a reference for the usual general statement of Fubini's theorem (Folland's Real Analysis, for instance) or for how the general statement implies this special case? It is literally just the general case applied with Lebesgue measure and counting measure.
    $endgroup$
    – Nate Eldredge
    Aug 15 '17 at 20:38











    25












    $begingroup$

    This is a theorem that will work:




    Theorem. If ${f_n}_n$ is a positive sequence of integrable functions and $f = sum_n f_n$ then
    $$int f = sum_n int f_n.$$




    Proof. Consider first two functions, $f_1$ and $f_2$. We can now find sequences ${phi_j}_j$ and ${psi_j}_j$ of (nonnegative) simple functions by a basic theorem from measure theory that increase to $f_1$ and $f_2$ respectively. Obviously $phi_j + psi_j uparrow f_1 + f_2$. We can do the same for any finite sum.



    Note that $int sum_1^N f_n = sum_1^N int f_n$ for any finite $N$. Now using the monotone convergence theorem we get



    $$sum int f_n = int f.$$



    Note 1: If you're talking about positive functions absolute convergence is the same as normal convergence as $|f_n| = f_n$.



    Note 2: Continuous functions will be certainly integrable if they have compact support or tend to $0$ fast enough as $x to pm infty$.






    share|cite|improve this answer











    $endgroup$













    • $begingroup$
      Just a little observation: you don't have say that $phi_juparrow f_1$ and $psi_juparrow f_2$.
      $endgroup$
      – leo
      Nov 19 '11 at 19:44












    • $begingroup$
      @leo Thanks! I have added this.
      $endgroup$
      – Jonas Teuwen
      Nov 19 '11 at 19:46










    • $begingroup$
      No problem Jonas
      $endgroup$
      – leo
      Nov 19 '11 at 19:55






    • 1




      $begingroup$
      @JonasTeuwen What does $phi_j uparrow f_1$ mean? I am not familiar with this notation.
      $endgroup$
      – Noir
      Jun 17 '18 at 17:30










    • $begingroup$
      It means that ${phi_j}$ is a monotonically non-decreasing sequence of functions which converges to $f_1$ pointwise.
      $endgroup$
      – Sean Haight
      Oct 31 '18 at 15:40
















    25












    $begingroup$

    This is a theorem that will work:




    Theorem. If ${f_n}_n$ is a positive sequence of integrable functions and $f = sum_n f_n$ then
    $$int f = sum_n int f_n.$$




    Proof. Consider first two functions, $f_1$ and $f_2$. We can now find sequences ${phi_j}_j$ and ${psi_j}_j$ of (nonnegative) simple functions by a basic theorem from measure theory that increase to $f_1$ and $f_2$ respectively. Obviously $phi_j + psi_j uparrow f_1 + f_2$. We can do the same for any finite sum.



    Note that $int sum_1^N f_n = sum_1^N int f_n$ for any finite $N$. Now using the monotone convergence theorem we get



    $$sum int f_n = int f.$$



    Note 1: If you're talking about positive functions absolute convergence is the same as normal convergence as $|f_n| = f_n$.



    Note 2: Continuous functions will be certainly integrable if they have compact support or tend to $0$ fast enough as $x to pm infty$.






    share|cite|improve this answer











    $endgroup$













    • $begingroup$
      Just a little observation: you don't have say that $phi_juparrow f_1$ and $psi_juparrow f_2$.
      $endgroup$
      – leo
      Nov 19 '11 at 19:44












    • $begingroup$
      @leo Thanks! I have added this.
      $endgroup$
      – Jonas Teuwen
      Nov 19 '11 at 19:46










    • $begingroup$
      No problem Jonas
      $endgroup$
      – leo
      Nov 19 '11 at 19:55






    • 1




      $begingroup$
      @JonasTeuwen What does $phi_j uparrow f_1$ mean? I am not familiar with this notation.
      $endgroup$
      – Noir
      Jun 17 '18 at 17:30










    • $begingroup$
      It means that ${phi_j}$ is a monotonically non-decreasing sequence of functions which converges to $f_1$ pointwise.
      $endgroup$
      – Sean Haight
      Oct 31 '18 at 15:40














    25












    25








    25





    $begingroup$

    This is a theorem that will work:




    Theorem. If ${f_n}_n$ is a positive sequence of integrable functions and $f = sum_n f_n$ then
    $$int f = sum_n int f_n.$$




    Proof. Consider first two functions, $f_1$ and $f_2$. We can now find sequences ${phi_j}_j$ and ${psi_j}_j$ of (nonnegative) simple functions by a basic theorem from measure theory that increase to $f_1$ and $f_2$ respectively. Obviously $phi_j + psi_j uparrow f_1 + f_2$. We can do the same for any finite sum.



    Note that $int sum_1^N f_n = sum_1^N int f_n$ for any finite $N$. Now using the monotone convergence theorem we get



    $$sum int f_n = int f.$$



    Note 1: If you're talking about positive functions absolute convergence is the same as normal convergence as $|f_n| = f_n$.



    Note 2: Continuous functions will be certainly integrable if they have compact support or tend to $0$ fast enough as $x to pm infty$.






    share|cite|improve this answer











    $endgroup$



    This is a theorem that will work:




    Theorem. If ${f_n}_n$ is a positive sequence of integrable functions and $f = sum_n f_n$ then
    $$int f = sum_n int f_n.$$




    Proof. Consider first two functions, $f_1$ and $f_2$. We can now find sequences ${phi_j}_j$ and ${psi_j}_j$ of (nonnegative) simple functions by a basic theorem from measure theory that increase to $f_1$ and $f_2$ respectively. Obviously $phi_j + psi_j uparrow f_1 + f_2$. We can do the same for any finite sum.



    Note that $int sum_1^N f_n = sum_1^N int f_n$ for any finite $N$. Now using the monotone convergence theorem we get



    $$sum int f_n = int f.$$



    Note 1: If you're talking about positive functions absolute convergence is the same as normal convergence as $|f_n| = f_n$.



    Note 2: Continuous functions will be certainly integrable if they have compact support or tend to $0$ fast enough as $x to pm infty$.







    share|cite|improve this answer














    share|cite|improve this answer



    share|cite|improve this answer








    edited Nov 19 '11 at 19:46

























    answered Nov 19 '11 at 19:24









    Jonas TeuwenJonas Teuwen

    9,53644676




    9,53644676












    • $begingroup$
      Just a little observation: you don't have say that $phi_juparrow f_1$ and $psi_juparrow f_2$.
      $endgroup$
      – leo
      Nov 19 '11 at 19:44












    • $begingroup$
      @leo Thanks! I have added this.
      $endgroup$
      – Jonas Teuwen
      Nov 19 '11 at 19:46










    • $begingroup$
      No problem Jonas
      $endgroup$
      – leo
      Nov 19 '11 at 19:55






    • 1




      $begingroup$
      @JonasTeuwen What does $phi_j uparrow f_1$ mean? I am not familiar with this notation.
      $endgroup$
      – Noir
      Jun 17 '18 at 17:30










    • $begingroup$
      It means that ${phi_j}$ is a monotonically non-decreasing sequence of functions which converges to $f_1$ pointwise.
      $endgroup$
      – Sean Haight
      Oct 31 '18 at 15:40


















    • $begingroup$
      Just a little observation: you don't have say that $phi_juparrow f_1$ and $psi_juparrow f_2$.
      $endgroup$
      – leo
      Nov 19 '11 at 19:44












    • $begingroup$
      @leo Thanks! I have added this.
      $endgroup$
      – Jonas Teuwen
      Nov 19 '11 at 19:46










    • $begingroup$
      No problem Jonas
      $endgroup$
      – leo
      Nov 19 '11 at 19:55






    • 1




      $begingroup$
      @JonasTeuwen What does $phi_j uparrow f_1$ mean? I am not familiar with this notation.
      $endgroup$
      – Noir
      Jun 17 '18 at 17:30










    • $begingroup$
      It means that ${phi_j}$ is a monotonically non-decreasing sequence of functions which converges to $f_1$ pointwise.
      $endgroup$
      – Sean Haight
      Oct 31 '18 at 15:40
















    $begingroup$
    Just a little observation: you don't have say that $phi_juparrow f_1$ and $psi_juparrow f_2$.
    $endgroup$
    – leo
    Nov 19 '11 at 19:44






    $begingroup$
    Just a little observation: you don't have say that $phi_juparrow f_1$ and $psi_juparrow f_2$.
    $endgroup$
    – leo
    Nov 19 '11 at 19:44














    $begingroup$
    @leo Thanks! I have added this.
    $endgroup$
    – Jonas Teuwen
    Nov 19 '11 at 19:46




    $begingroup$
    @leo Thanks! I have added this.
    $endgroup$
    – Jonas Teuwen
    Nov 19 '11 at 19:46












    $begingroup$
    No problem Jonas
    $endgroup$
    – leo
    Nov 19 '11 at 19:55




    $begingroup$
    No problem Jonas
    $endgroup$
    – leo
    Nov 19 '11 at 19:55




    1




    1




    $begingroup$
    @JonasTeuwen What does $phi_j uparrow f_1$ mean? I am not familiar with this notation.
    $endgroup$
    – Noir
    Jun 17 '18 at 17:30




    $begingroup$
    @JonasTeuwen What does $phi_j uparrow f_1$ mean? I am not familiar with this notation.
    $endgroup$
    – Noir
    Jun 17 '18 at 17:30












    $begingroup$
    It means that ${phi_j}$ is a monotonically non-decreasing sequence of functions which converges to $f_1$ pointwise.
    $endgroup$
    – Sean Haight
    Oct 31 '18 at 15:40




    $begingroup$
    It means that ${phi_j}$ is a monotonically non-decreasing sequence of functions which converges to $f_1$ pointwise.
    $endgroup$
    – Sean Haight
    Oct 31 '18 at 15:40











    0












    $begingroup$

    While most of the time I would use the Fubini/Tonelli conditions, the dominated convergence theorem is actually strictly stronger in this mixed sum/integral case, because it can take into account the order structure of the integers. An example (that I first worked up back in 2009:



    Consider the calculation
    begin{align*}ln 2 &= int_0^1 frac1{1+x},dx = int_0^1sum_{n=0}^{infty} (-1)^n x^n,dx\
    ?&= sum_{n=0}^{infty}int_0^1(-1)^n x^n,dx = 1-frac12+frac13-frac14+cdotsend{align*}

    Fubini's theorem isn't strong enough to justify the interchange. If we put absolute values on the terms, it blows up to $int_0^1 frac1{1-x},dx = 1+frac12+frac13+frac14+cdots=infty$.



    On the other hand, the dominated convergence theorem cares about the partial sums $sum_{n=0}^{N}(-1)^n x^n$. By the alternating series estimate,
    $$0le sum_{n=0}^{N}(-1)^n x^nle 1$$
    for all $xin [0,1]$. $1$ is integrable on this interval, and the interchange
    $$int_0^1left(lim_{Ntoinfty}sum_{n=0}^{N}(-1)^n x^nright),dx = lim_{Ntoinfty}int_0^1 sum_{n=0}^{N}(-1)^n x^n,dx$$
    is justified, proving the result $1-frac12+frac13-frac14+cdots=ln 2$.



    This situation with the dominated convergence theorem being stronger than Fubini's theorem can come up when we've got a reasonable bound on partial sums but not absolute convergence as a whole.

    The monotone convergence theorem, on the other hand, is exactly the same as Tonelli's theorem - when everything's positive, either both sides are the same and finite or both sides are infinite.






    share|cite|improve this answer









    $endgroup$


















      0












      $begingroup$

      While most of the time I would use the Fubini/Tonelli conditions, the dominated convergence theorem is actually strictly stronger in this mixed sum/integral case, because it can take into account the order structure of the integers. An example (that I first worked up back in 2009:



      Consider the calculation
      begin{align*}ln 2 &= int_0^1 frac1{1+x},dx = int_0^1sum_{n=0}^{infty} (-1)^n x^n,dx\
      ?&= sum_{n=0}^{infty}int_0^1(-1)^n x^n,dx = 1-frac12+frac13-frac14+cdotsend{align*}

      Fubini's theorem isn't strong enough to justify the interchange. If we put absolute values on the terms, it blows up to $int_0^1 frac1{1-x},dx = 1+frac12+frac13+frac14+cdots=infty$.



      On the other hand, the dominated convergence theorem cares about the partial sums $sum_{n=0}^{N}(-1)^n x^n$. By the alternating series estimate,
      $$0le sum_{n=0}^{N}(-1)^n x^nle 1$$
      for all $xin [0,1]$. $1$ is integrable on this interval, and the interchange
      $$int_0^1left(lim_{Ntoinfty}sum_{n=0}^{N}(-1)^n x^nright),dx = lim_{Ntoinfty}int_0^1 sum_{n=0}^{N}(-1)^n x^n,dx$$
      is justified, proving the result $1-frac12+frac13-frac14+cdots=ln 2$.



      This situation with the dominated convergence theorem being stronger than Fubini's theorem can come up when we've got a reasonable bound on partial sums but not absolute convergence as a whole.

      The monotone convergence theorem, on the other hand, is exactly the same as Tonelli's theorem - when everything's positive, either both sides are the same and finite or both sides are infinite.






      share|cite|improve this answer









      $endgroup$
















        0












        0








        0





        $begingroup$

        While most of the time I would use the Fubini/Tonelli conditions, the dominated convergence theorem is actually strictly stronger in this mixed sum/integral case, because it can take into account the order structure of the integers. An example (that I first worked up back in 2009:



        Consider the calculation
        begin{align*}ln 2 &= int_0^1 frac1{1+x},dx = int_0^1sum_{n=0}^{infty} (-1)^n x^n,dx\
        ?&= sum_{n=0}^{infty}int_0^1(-1)^n x^n,dx = 1-frac12+frac13-frac14+cdotsend{align*}

        Fubini's theorem isn't strong enough to justify the interchange. If we put absolute values on the terms, it blows up to $int_0^1 frac1{1-x},dx = 1+frac12+frac13+frac14+cdots=infty$.



        On the other hand, the dominated convergence theorem cares about the partial sums $sum_{n=0}^{N}(-1)^n x^n$. By the alternating series estimate,
        $$0le sum_{n=0}^{N}(-1)^n x^nle 1$$
        for all $xin [0,1]$. $1$ is integrable on this interval, and the interchange
        $$int_0^1left(lim_{Ntoinfty}sum_{n=0}^{N}(-1)^n x^nright),dx = lim_{Ntoinfty}int_0^1 sum_{n=0}^{N}(-1)^n x^n,dx$$
        is justified, proving the result $1-frac12+frac13-frac14+cdots=ln 2$.



        This situation with the dominated convergence theorem being stronger than Fubini's theorem can come up when we've got a reasonable bound on partial sums but not absolute convergence as a whole.

        The monotone convergence theorem, on the other hand, is exactly the same as Tonelli's theorem - when everything's positive, either both sides are the same and finite or both sides are infinite.






        share|cite|improve this answer









        $endgroup$



        While most of the time I would use the Fubini/Tonelli conditions, the dominated convergence theorem is actually strictly stronger in this mixed sum/integral case, because it can take into account the order structure of the integers. An example (that I first worked up back in 2009:



        Consider the calculation
        begin{align*}ln 2 &= int_0^1 frac1{1+x},dx = int_0^1sum_{n=0}^{infty} (-1)^n x^n,dx\
        ?&= sum_{n=0}^{infty}int_0^1(-1)^n x^n,dx = 1-frac12+frac13-frac14+cdotsend{align*}

        Fubini's theorem isn't strong enough to justify the interchange. If we put absolute values on the terms, it blows up to $int_0^1 frac1{1-x},dx = 1+frac12+frac13+frac14+cdots=infty$.



        On the other hand, the dominated convergence theorem cares about the partial sums $sum_{n=0}^{N}(-1)^n x^n$. By the alternating series estimate,
        $$0le sum_{n=0}^{N}(-1)^n x^nle 1$$
        for all $xin [0,1]$. $1$ is integrable on this interval, and the interchange
        $$int_0^1left(lim_{Ntoinfty}sum_{n=0}^{N}(-1)^n x^nright),dx = lim_{Ntoinfty}int_0^1 sum_{n=0}^{N}(-1)^n x^n,dx$$
        is justified, proving the result $1-frac12+frac13-frac14+cdots=ln 2$.



        This situation with the dominated convergence theorem being stronger than Fubini's theorem can come up when we've got a reasonable bound on partial sums but not absolute convergence as a whole.

        The monotone convergence theorem, on the other hand, is exactly the same as Tonelli's theorem - when everything's positive, either both sides are the same and finite or both sides are infinite.







        share|cite|improve this answer












        share|cite|improve this answer



        share|cite|improve this answer










        answered Dec 30 '18 at 11:19









        jmerryjmerry

        3,592514




        3,592514






























            draft saved

            draft discarded




















































            Thanks for contributing an answer to Mathematics Stack Exchange!


            • Please be sure to answer the question. Provide details and share your research!

            But avoid



            • Asking for help, clarification, or responding to other answers.

            • Making statements based on opinion; back them up with references or personal experience.


            Use MathJax to format equations. MathJax reference.


            To learn more, see our tips on writing great answers.




            draft saved


            draft discarded














            StackExchange.ready(
            function () {
            StackExchange.openid.initPostLogin('.new-post-login', 'https%3a%2f%2fmath.stackexchange.com%2fquestions%2f83721%2fwhen-can-a-sum-and-integral-be-interchanged%23new-answer', 'question_page');
            }
            );

            Post as a guest















            Required, but never shown





















































            Required, but never shown














            Required, but never shown












            Required, but never shown







            Required, but never shown

































            Required, but never shown














            Required, but never shown












            Required, but never shown







            Required, but never shown







            Popular posts from this blog

            Human spaceflight

            Can not write log (Is /dev/pts mounted?) - openpty in Ubuntu-on-Windows?

            張江高科駅